Extreme value theorem question?

Click For Summary
The discussion centers on the Extreme Value Theorem (EVT) and its application to the function g(x) = 1/(x-2), which is continuous on the interval [3, ∞). It is clarified that g(x) does not attain a minimum value on this interval, but this does not contradict the EVT because [3, ∞) is not a closed interval. The EVT applies to continuous functions defined on closed and bounded intervals, and since [3, ∞) is unbounded, it does not meet the criteria. The conversation emphasizes the need for precise terminology regarding intervals in relation to the EVT. Overall, the function's behavior aligns with the theorem's stipulations.
indigo1
Messages
3
Reaction score
0

Homework Statement



Given g(x) = 1/x-2 is continuous and defined on [3, infinity). g(x) has no minimum value on the interval [3, infinity ). Does this function contradict the EVT? Explain.



The Attempt at a Solution



was wondering what would be a better explanation for this one?

No, because the the extreme value theorems says that a continuous function defined on a DEFINITE interval will attain a max/min in the interval. [3, ∞) is not a definite interval. [3, a] where a is some number greater than 3 is and would achieve its minimum at x = a.

or

g has no minimum on this interval even though its continuous. It does not contradict how ever because the interval is not open.
 
Physics news on Phys.org
I don't think the EVT says anything about DEFINITE intervals, nor does it says anything about open intervals. What kind of intervals does it talk about?
 
indigo1 said:
No, because the the extreme value theorems says that a continuous function defined on a DEFINITE interval will attain a max/min in the interval. [3, ∞) is not a definite interval. [3, a] where a is some number greater than 3 is and would achieve its minimum at x = a.

The thought process behind this answer seems appropriate and correct. :smile:

Though, there is no 'definite interval'. You need to rephrase that word looking back at the definition of EVT.
 
Question: A clock's minute hand has length 4 and its hour hand has length 3. What is the distance between the tips at the moment when it is increasing most rapidly?(Putnam Exam Question) Answer: Making assumption that both the hands moves at constant angular velocities, the answer is ## \sqrt{7} .## But don't you think this assumption is somewhat doubtful and wrong?

Similar threads

Replies
3
Views
2K
  • · Replies 12 ·
Replies
12
Views
2K
  • · Replies 14 ·
Replies
14
Views
2K
  • · Replies 3 ·
Replies
3
Views
2K
  • · Replies 2 ·
Replies
2
Views
1K
  • · Replies 26 ·
Replies
26
Views
3K
Replies
4
Views
2K
Replies
11
Views
2K
  • · Replies 1 ·
Replies
1
Views
2K
  • · Replies 1 ·
Replies
1
Views
2K